Breast Augmentation, Mastopexy Flashcards
A 35-year-old woman with tuberous breast deformity is scheduled to undergo augmentation/mastopexy. A smooth, round, cohesive gel implant will be used. This patient is at higher risk for which of the following complications when compared with augmentation/mastopexy performed on a patient without a tuberous breast?
A) Capsule contracture
B) Double bubble
C) Hematoma
D) Nipple-areola depigmentation
E) Rippling
The correct response is Option B.
The classic features of a tuberous breast deformity include a constricted base with a high inframammary crease and herniation of breast parenchyma into the nipple-areola complex producing a large-diameter areola. Variable extent of micromastia is associated as well as breast asymmetry. When a patient has a high and tight inframammary crease, this crease must be released to accommodate an implant and allow correction of the deformity. If this native crease does not fully expand, then a double bubble will occur. Over time, the lower pole skin stretches in response to the implant and this double bubble often improves spontaneously. The incidence of capsule contracture, hematoma, nipple-areola depigmentation, and rippling should be similar to a patient who undergoes periareolar augmentation/mastopexy without a tuberous breast.
A 36-year-old woman is evaluated because of spontaneous galactorrhea 6 days after undergoing augmentation mammaplasty. Which of the following factors most likely contributed to this outcome?
A) Inframammary placement of the incision
B) Subglandular versus dual-plane position of the device
C) Surgical interruption of the intercostal nerves
D) Use of silicone versus saline breast implants
The correct response is Option C.
Although no one knows exactly what leads to postoperative galactorrhea, it is observed to occur more often in parous women and theorized to occur due to a combination of factors which simulate suckling or change in the innervation of the chest wall and nipple-areola complex. This would include increased tissue pressure related to the implant placement and interruption of intercostal nerves. No relationship has been identified between incision placement (peri-areolar, inframammary, transaxillary, or even peri-thelial) and postoperative galactorrhea. Similarly no relationship has been identified between device positioning (dual-plane, subglandular, and submuscular) and postoperative galactorrhea. Again, no relationship has been observed in implant type, saline versus silicone, and postoperative galactorrhea.
A 45-year-old woman comes to the office for consultation regarding augmentation mammaplasty. She wears a size 32B brassiere; height is 5 ft 3 in (160 cm), and weight is 130 lb (59 kg). Subglandular placement of saline prostheses is planned. Which of the following is the primary advantage of using saline rather than silicone prostheses in this patient?
A)Easier detection of rupture
BLess capsular formation
C)Less wrinkling
D)Lighter prosthesis
E)Lower risk of leakage
The correct response is Option A.
Although both silicone and saline prostheses rupture at a similar rate, a saline rupture is more easily detectable because the saline is resorbed in the body. The deflated breast will be smaller in volume. Subtle changes, such as decreased upper pole fullness or increased softness, may be the only clues to silicone rupture on physical examination. Ultrasonography or MRI may be needed to confirm the diagnosis.
Saline prostheses are firmer than silicone; they are more likely to be palpable than silicone prostheses as well. Neither prosthesis has been associated with systemic immune syndromes, and both prostheses produce capsular contracture, wrinkling, and leakage.
Which of the following is the most common complication of periareolar mastopexy?
(A) Dehiscence
(B) Excessive breast projection
(C) Nipple discharge
(D) Recurrent ptosis
(E) Widening of the areola
The correct response is Option E.
Widening of the areola is the most common complication following periareolar mastopexy. Techniques developed to minimize the occurrence of areolar dilation include the use of nonresorbable purse-string sutures and creation of an excessively small areola at the time of surgery to compensate for postoperative widening.
Less common complications include dehiscence and recurrent ptosis. Excessive projection is rarely seen with periareolar mastopexy; flattened or globular breast shapes are more commonly reported. Nipple discharge is not associated with mastopexy.
A 26-year-old woman comes to the office for consultation regarding right mammary hypoplasia and a superiorly displaced nipple-areola complex. Examination shows a depressed right chest wall. The pectoralis major muscle is anatomically normal. Which of the following is the most likely diagnosis?
A ) Anterior thoracic hypoplasia
B ) Pectus carinatum
C ) Pectus excavatum
D ) Poland syndrome
E ) Sternal cleft
The correct response is Option A.
The most likely diagnosis in this patient is anterior thoracic hypoplasia. Anterior thoracic hypoplasia is a syndrome composed of an anterior chest wall depression resulting from posteriorly displaced ribs, hypoplasia of the ipsilateral breast, and a superiorly displaced nipple-areola complex. The sternum is in normal position, and the pectoralis major muscle is normal.
Pectus excavatum is the most common congenital chest wall abnormality in which the ribs and sternum form abnormally, resulting in a concave anterior chest wall. Typically, the lower third of the sternum is involved. In the most severe form, pectus excavatum can present with the sternum adjacent to the vertebral bodies associated with cardiopulmonary abnormalities. In contrast, pectus carinatum is a chest wall deformity in which the sternum and ribs are forced anteriorly, creating the appearance of a €œpigeon €™s chest. € Pectus excavatum and carinatum have sternal involvement, but they do not involve changes in the development of the breast.
Poland syndrome is a congenital anomaly characterized by a number of unilateral findings. The classic features of Poland syndrome include absence of the sternal head of the pectoralis major, hypoplasia and/or aplasia of the breast or nipple, deficiency of subcutaneous fat and axillary hair, abnormalities of the rib cage, and upper extremity anomalies. In its simplest form, Poland syndrome may present with only mild hypoplasia of the breast and lateral displacement of the nipple. Complex presentations of Poland syndrome include hypoplasia or aplasia of the chest wall musculature (serratus, external oblique, pectoralis minor, and latissimus dorsi muscles) or total absence of the anterolateral ribs with herniation of the lung.
Sternal cleft is a rare congenital defect of the anterior chest wall resulting from a failure of midline fusion of the sternum. Depending on the degree of clefting, there are complete and incomplete forms. The sternal cleft is clinically significant because of the potential for the lack of protection to the heart and great vessels. Sternal clefts are not associated with aplasia or hypoplasia of the breast.
A 35-year-old woman comes to the office for consultation regarding augmentation mammaplasty. A preoperative mammogram is most indicated if the patient’s history includes which of the following?
A ) A grandmother diagnosed with breast cancer at age 73 years
B ) A mother diagnosed with breast cancer at age 45 years
C ) Personal history of breast cysts
D ) Personal history of fibroadenoma
E ) A sister diagnosed with ovarian cancer
The correct response is Option B.
Among the risk factors for breast cancer, family history is the most significant. It can be divided into two broad categories: familial breast cancer, which most likely results from changes in multiple low penetrance genes coupled with environmental influences, and hereditary breast cancer, which results in high penetrance mutation in a single gene.
Familial breast cancer is relatively common and conveys a modest elevation in risk compared with genetic breast cancer, which is rare but associated with high risk.
A family history of breast cancer has been demonstrated to increase the risk of breast cancer in multiple studies. Breast cancer in a first-degree relative increases the risk of breast cancer, and that risk decreases as the age of the affected relative increases (ie, it is a 2.3 relative risk factor if the affected relative is under 50 years of age; it is 1.8 if she is over 50). Individuals whose first-degree relatives have bilateral breast cancer have an increased risk of 5.5 times the normal population.
A 16-year-old girl is referred to the clinic by her pediatrician for correction of breast asymmetry. The patient reports that her right breast has always been smaller. Physical examination shows mildly shortened right fingers and a Tanner III right breast. Examination shows that the left breast is Tanner IV and no masses. Which of the following is the Mathes and Nahai Classification of the muscle most likely involved in this congenital disorder?
A) Type I
B) Type II
C) Type III
D) Type IV
E) Type V
The correct response is Option E.
The pectoralis major has a dual blood supply from both a dominant primary pedicle (pectoral branches of the thoracoacromial artery) and secondary segmental perforators (internal mammary/thoracic perforators).
The absence of the pectoralis major muscle and associated hand deformity is pathognomonic for Poland syndrome. The etiology is unclear but suspected due to vascular interruption during embryogenesis. The diagnosis is often delayed until puberty when asymmetric breast development is noted. Brachydactyly, syndactyly, or ectrodactyly are common and can present with various severity.
Type I flaps have a single dominant blood supply (e.g., rectus femoris or gastrocnemius muscles). Type II flaps have a dominant and minor vascular pedicle (e.g., gracilis or soleus muscles). Type III flaps have 2 dominant pedicles (e.g., pectoralis minor, rectus abdominis and serratus muscles). Type IV flaps only have segmental blood supply (e.g., sartorius or tibialis anterior muscles).
A 36-year-old woman comes to the office for consultation regarding €œsagging € breasts 10 years after undergoing bilateral augmentation mammaplasty with subglandular placement of saline prostheses. Physical examination shows Grade 2 ptosis and an axillary scar. A mastopexy with capsulotomy and replacement of prostheses is planned. Which of the following pedicles is LEAST likely to preserve the blood supply to the nipple-areola complex?
A ) Inferior
B ) Medial
C ) Superior
D ) Superolateral
E ) Superomedial
The correct response is Option A.
Secondary mastopexy in the augmented patient can be particularly hazardous. In addition to scars from prior surgery, the soft-tissue envelope surrounding the prosthesis frequently becomes attenuated. Tebbetts observed that the €œconsequences of excessively large breast implants include ptosis, tissue stretching, tissue thinning, inadequate soft-tissue cover, [and] …subcutaneous tissue atrophy. € Gravity causes most soft-tissue thinning and atrophy to eventually occur in the inferior pole of the augmented breast. Therefore, in secondary mastopexy augmentation procedures, blood supply to the nipple-areola complex should generally rely on a medial, superior, superomedial, or superolateral pedicle.
A 23-year-old woman comes to the office because she is interested in augmentation mammaplasty. Physical examination shows a right-sided sunken anterior chest wall, hypoplasia of the right breast with a superiorly placed nipple-areola complex, normal pectoralis muscle, and normal sternal position. This patient most likely has which of the following congenital deformities?
A) Amastia
B) Anterior thoracic hypoplasia
C) Pectus carinatum
D) Pectus excavatum
E) Poland syndrome
The correct response is Option B.
Amastia refers to an uncommon developmental condition in which the breast and nipple are absent. Some women are immediately given the diagnosis of Poland syndrome or pectus deformity when they exhibit abnormalities of the anterior chest wall. Poland syndrome involves an abnormal pectoralis muscle while pectus deformities do not. Pectus deformities involve alteration in the appearance or location of the sternum and its costal attachments. Another less commonly realized diagnosis is that of anterior thoracic hypoplasia in which patients share the same characteristics of unilateral sunken anterior chest wall, hypoplasia of the breast, superiorly placed nipple-areola complex, normal pectoralis muscle, and normal sternal position.
Which of the following proteins has been implicated in the pathogenesis of breast implant capsule formation?
(A) Albumin
(B) Fibrinogen
(C) Complement
(D) IgG
The correct response is Option B.
The surface-bound protein fibrinogen has been implicated in the generation of inflammatory responses to biomaterials (ie, implants); early protein absorption of these biomaterials mediates the foreign body response. Understanding the intricate pathways that result in fibrinogen absorption and its subsequent inflammatory response, leading to capsule formation, may aid in the prevention and management of breast implant capsular contracture.
Other dominant proteins in the body, such as albumin, complement, and immunoglobulin G (IgG), have not been shown to play a critical role in capsule formation resulting from foreign body reaction.
A 26-year-old woman who underwent augmentation mammaplasty six months ago comes to the office because she has numbness of the right nipple. The most likely cause is injury to which of the following intercostal nerves?
A ) Second
B ) Third
C ) Fourth
D ) Fifth
E ) Sixth
The correct response is Option C.
According to Courtiss and Goldwyn, the fourth intercostal nerve is the most important nipple innervator.
The anterior cutaneous branches of the second through sixth intercostal nerves provide the medial innervation. The anterior rami of the lateral cutaneous branches of the third through sixth intercostal nerves provide the lateral innervation.
It has been demonstrated that the lateral cutaneous branches of the third through fifth intercostal nerves and the anterior cutaneous branches of the second through fifth intercostal nerves all contribute to nipple supply. The lateral cutaneous branch of the fourth intercostal nerve has been traced into the nipple and found to have two branches. The deep branch passes inferolaterally on the pectoralis major fascia before coursing up into the areola, whereas the superficial branch passes up through the superficial parenchyma.
Prior to breast augmentation, management of milky discharge in a regularly menstruating woman should include which of the following?
(A) Observation
(B) Massage
(C) Measurement of serum prolactin level
(D) Administration of antibiotics
(E) Ovarian biopsy
The correct response is Option C.
Although breast discharge is rare in regularly menstruating women who have never been pregnant, it has been shown to occur in 25% of women who have been pregnant in the past. Complete evaluation of galactorrhea should include measurement of the serum level of prolactin (a lactogenic hormone required for milk production), thyroid function studies to rule out hypothyroidism, and a history of all medications, as tricyclic antidepressants and fluoxetine have been shown to contribute to breast discharge. Women who have increased serum prolactin levels should then undergo MRI evaluation to rule of the possibility of pituitary tumor. According to a series of four studies involving more than 500 patients with galactorrhea, a pituitary tumor was the underlying cause in 25%; in contrast, 50% of those studied had idiopathic causes. Appropriate management of idiopathic galactorrhea includes administration of bromocriptine to suppress the release of prolactin.
Observation is inadequate management because of the risk for pituitary tumor in these patients. Breast massage is not appropriate and will instead maintain or even initiate galactorrhea in women with prior pregnancies. Because galactorrhea is not associated with infection, antibiotics should not be administered; however, if the discharge is bloody or has brown or green discoloration, the patient should be evaluated for possible infection or tumor. Ovarian biopsy is only indicated if evaluation shows ovarian pathology.
A 30-year-old woman undergoes augmentation mammaplasty with silicone gel prostheses. During the procedure, smooth prostheses are positioned subglandularly. The subglandular placement increases this patient=s risk of which of the following complications?
(A) Capsular contracture
(B) Double-bubble appearance
(C) Infection of the implant
(D) Rippling of the implant
(E) Rupture of the implant
The correct response is Option A.
Capsular contracture remains one of the main drawbacks to the use of silicone breast prostheses. Submuscular placement is a well-established method of reducing the rate of contracture. The introduction of implant-surface texturing in the late 1980s has greatly reduced the contracture rate for prostheses placed subglandularly.
Development of capsular contracture is clearly more common in the first two years after subglandular implantation, regardless of the implant type. The large difference in the rate of contracture between textured and smooth prostheses in the subglandular position seems to be negligible in subpectoral placement; both types of implant have low contracture rates.
The causes of capsular contracture and the effect of surface texturing and implant position in reducing its incidence are still not clear. Capsule formation is a normal response to the introduction of foreign material and, like most physiologic responses, varies by degree and timing. Additionally, there are general patient factors and local breast factors. The literature suggests that capsule response may be altered by other factors such as infection, diffusion of silicone gel, and smoking.
The introduction of surface texturing alters the capsule response. Texturing may produce a more disorganized collagen pattern in the capsule. It has been suggested that subpectoral prostheses have a lower rate of capsular contracture, regardless of surface texturing, because of the massaging action of the overlying pectoralis major.
A 47-year-old woman is referred by her primary care physician to evaluate a suspected intracapsular rupture of her prosthesis on the left identified during routine mammography. She underwent primary augmentation mammaplasty with subglandular placement of single-lumen silicone breast prostheses in 1990. Physical examination shows a smaller breast on the left. An MRI is requested. Which of the following findings on MRI is most likely to confirm the diagnosis?
A ) Double wall sign
B ) Linguine sign
C ) Multiple echogenic lines
D ) Reverse double-lumen sign
E ) Snowstorm sign
The correct response is Option B.
MRI, mammography, ultrasonography, and CT scanning have all been used to diagnose silicone breast prosthesis rupture.
Although each modality has specific strengths and weaknesses that may make a particular modality the study of choice for an individual patient, MRI of silicone breast prostheses reports the highest sensitivity and specificity for detection of silicone prosthesis rupture.
Of the options listed, only the linguine sign is consistent with intracapsular silicone prosthesis rupture and represents the prosthesis shell floating in free silicone gel.
The double wall sign, also known as Rigler sign, is a radiographic sign of pneumoperitoneum.
Snowstorm sign and echogenic lines may be seen on ultrasound examination.
Water suppression or a reverse double-lumen sign would not be expected findings in a single-lumen device but may have a role in double-lumen devices.
A 53-year-old woman comes to the office because of unilateral swelling of the breast 5 years after undergoing subglandular augmentation mammaplasty. A diagnosis of anaplastic large T-cell lymphoma (ALCL) is established. Which of the following is most likely to represent the progression of this patient’s disease when compared with a patient who has ALCL but no breast prostheses?
A) A more aggressive clinical course and a poorer prognosis
B) A more aggressive clinical course but a more favorable prognosis
C) A more indolent clinical course and a more favorable prognosis
D) A more indolent clinical course but a poorer prognosis
E) The same clinical course and prognosis
The correct response is Option C.
Anaplastic large T-cell lymphoma (ALCL) is a rare (1 per million) non-Hodgkin lymphoma that has been reported in women with and without breast prostheses. However, increasing case reports suggest an association with breast prostheses, although direct causation has not been established. ALCL associated with breast prostheses has malignant cells infiltrating the periprosthetic capsule or in the periprosthetic fluid collection. It is associated with both silicone- and saline-filled prostheses and seen in patients who have had prostheses for augmentation mammaplasty as well as breast reconstruction. Although the cytology is the same between ALCL associated with and without breast prostheses, ALCL that develops around prostheses tend to have an indolent clinical course and favorable prognosis when compared with systemic ALCL.
A 36-year-old woman is being evaluated 17 years after undergoing augmentation mammaplasty with silicone gel implants. On examination, the implants are soft and minimally palpable; she reports no complications. This patient is at risk for which of the following?
(A) Implant rupture
(B) Increased silicon levels in breast milk
(C) Rheumatoid arthritis
(D) Scleroderma
(E) Silicone synovitis
The correct response is Option A.
This 36-year-old woman is at risk for implant rupture, which has been shown in recent studies to increase proportionately with the age of the implant. One retrospective study determined the mean age of implant rupture to be 13.4 years. MRI is most effective for assessing potential implant rupture, which in one study was reported in as many as 71% of implant patients. Another study showed that 50% of patients who had had implants for seven to 10 years showed evidence of rupture or hemorrhage on MRI.
Although attempts have been made to associate silicone gel implants to the onset of rheumatoid symptoms in children who were breast-fed, one study showed no difference in silicone levels measured in breast milk in women with implants versus controls. In addition, several large epidemiologic studies have shown no link between silicone gel implants and the subsequent development of either rheumatologic (ie, rheumatoid arthritis) or connective tissue (ie, scleroderma) diseases. Silicone synovitis occurs in patients who have silicone joint prostheses, but not in patients with silicone gel breast implants.
Which of the following is the most likely site of ectopic breast tissue in a patient with ectopic polymastia?
A ) Axilla
B ) Costal margin
C ) Dorsal thigh
D ) Pubis
E ) Vulva
The correct response is Option C.
Polymastic breast tissue can be categorized either as accessory or ectopic. Ectopic breast tissue is found outside the milk line at such sites as the scalp, ear, back, shoulder, epigastrium, and posterior or dorsal thigh.
Accessory polymastia occurs along the milk line. Greater than 90% of accessory breast tissue is localized to the chest region. The axilla, groin, vulva, and medial thigh may also be affected as well as regions above or below the normal breast, such as the costal margin.
Which of the following is the most common complication associated with “donut” mastopexy?
A) Boxy breast shape
B) Increased distance from nipple to inframammary fold
C) Loss of nipple sensation
D) Nipple necrosis
E) Widening of the areola
The correct response is Option E.
A common complication of the “donut” (circumareolar) mastopexy is widening of the areola. This can be minimized by using a Gore-Tex suture placed using the “wagon-wheel” technique and limiting the amount of skin resected to a 2:1 ratio of outside diameter to areolar diameter.
Boxy breast shape is associated with Wise pattern mastopexy. Nipple necrosis is associated with combined augmentation and mastopexy. Increased distance from the nipple to the inframammary fold is associated with vertical mastopexies in which the height of the medial and lateral pillars is too tall. Loss of nipple sensitivity is unusual because there is no parenchymal resection.
A 25-year-old woman comes to the office because she has a lump in her right armpit. She reports that the lump increases in size and becomes tender during her period. She also says that it restricts arm movement and interferes with her ability to play tennis, especially during menses. Examination shows a 4 * 4-cm, soft, mobile mass in the right axilla that is tender to palpation. There is no evidence of firmness or palpable nodules within the mass. Which of the following is the most appropriate next step in management?
A ) Excision of axillary tissue
B ) Fine-needle aspiration
C ) Incisional biopsy
D ) Mammogram
E ) Sentinel lymph node biopsy
The correct response is Option A.
Axillary accessory breast tissue should be removed surgically. It is found in 0.4% to 6% of women and may be asymptomatic, cause pain, restrict arm movement, or cause cosmetic problems or anxiety. There have been reports of malignant degeneration of this accessory breast tissue, and the current recommendations are for simple excision through an axillary incision.
Fine-needle aspiration may diagnose the presence of breast tissue, but it also may be inconclusive. A mammogram would not be helpful in confirmation of this diagnosis; however, MRI has been used to evaluate the presence of axillary breast tissue. A sentinel lymph node biopsy would not be necessary given the lack of malignancy. An incisional biopsy taking a sample of tissue is unnecessary, as the lesion should be completely excised.
A 24-year-old woman is undergoing endoscopic transaxillary augmentation mammaplasty. Which of the following is most appropriate to preserve sensation in the medial aspect of the upper extremity?
A ) Avoiding dissection into the axillary fat
B ) Blunt dissection near the clavicle
C ) Identification of the sensory nerves within the axilla
D ) Positioning of the prosthesis subpectorally
E ) Preservation of the lateral pectoral nerve
The correct response is Option A.
During transaxillary augmentation mammaplasty, prevention of sensory changes to the medial aspect of the upper extremity requires a subdermal dissection and avoids dissection into the axillary fat. Branches of the intercostobrachial and medial brachial cutaneous nerves provide sensory innervation to the medial upper extremity. Both nerves course superficially through the axillary fat posterior to the lateral border of the pectoralis major muscle. Dissection within the axillary fat risks injury to these nerves with subsequent anesthesia or paresthesia of the inner arm.
Identification of the nerves within the axilla requires dissection into axillary fat and risks injury to the sensory nerves. Sensory innervation to the medial aspect of the upper extremity is not affected by the positioning of the prosthesis (subpectoral versus subglandular) or dissection near the clavicle. The lateral pectoral nerve provides motor innervation to the lower third of the pectoralis major muscle.
A 65-year-old woman comes to the office 1 month before a scheduled mastopexy. Annual mammography shows a 1.5-cm mass in the upper outer quadrant. Core needle biopsy is performed. Pathologic examination of excised tissue identifies papilloma without atypia. Which of the following is the most appropriate next step in management?
A) Bilateral breast sonography
B) Excisional biopsy of needle-localized area
C) Repeat annual mammography in 12 months
D) Repeat mammography at 6-month intervals for 1 year
E) Stereotactic vacuum-assisted biopsy
The correct response is Option B.
Percutaneous biopsy methods are commonly accepted for the initial evaluation of clinically occult breast lesions, although certain nonmalignant lesions pose dilemmas with respect to the most appropriate clinical management. Papillary lesions of the breast can either be benign or malignant, although differentiation is radiologically difficult. Moreover, it is difficult for pathologists to reliably distinguish among benign, atypical, and malignant papillary lesions on the limited fragmented tissue specimens they receive after needle sampling.
Previous studies have demonstrated high rates of ductal carcinoma in situ (11%) in patients diagnosed with benign papillomas by needle biopsy and who subsequently underwent a surgical excision, although conflicting data suggest an extremely decreased rate of malignancy when histology is benign on needle biopsy.
The management of benign papillary lesions is somewhat controversial. Although conservative follow-up with either yearly mammogram or short-interval follow-up may be appropriate for certain patients diagnosed with benign papilloma, certain features of this patient’s lesion make conservative follow-up inappropriate. Sonographic follow-up in a 65-year-old woman with mature breast parenchyma and a solid mammographically detected mass would not provide much additional information, and a repeat percutaneous biopsy, whether core needle or vacuum-assisted, would also not be effective. Given the size of the lesion and the age of the patient, surgical excision is warranted despite the lack of atypia on needle biopsy. Benign papillomas tend to be smaller than 1 cm and centrally located, whereas malignant lesions are more often greater than 1.5 cm and are peripherally located.
A 25-year-old woman comes to the office because she is dissatisfied after undergoing breast augmentation mammaplasty for correction of tuberous breast deformities. Physical examination shows two parallel creases running transversely across the lower pole of each breast with inferior displacement of the implant. Which of the following best describes the position of the original inframammary fold in this patient?
A) Above the superior and inferior transverse creases
B) At the inferior transverse crease
C) At the superior transverse crease
D) Below the superior and inferior transverse creases
The correct response is Option C.
A double-bubble breast deformity following breast augmentation mammaplasty is represented by the development of two parallel, curvilinear transverse lines in the lower pole of the breast. The native inframammary fold is disrupted and represented by the superior transverse line. The lower transverse line represents the lower limit of implant pocket dissection or the final position of implant descent.
Predisposing anatomic factors for the development of a double-bubble deformity include tuberous breasts, constricted inframammary folds, or a short inframammary fold-to-nipple distance. Other factors that can increase the risk for the development of a double-bubble deformity include glandular ptosis, postpartum involution of the breasts, excessive implant size, and overdissection of the implant pocket. Correction of the double-bubble deformity may require conversion of the implant to a subglandular position, capsulorrhaphies, use of form-stable implants, or dermal grafts.
A 35-year-old woman, gravida 2, para 2, seeks implant-based augmentation mammaplasty. She breastfed both her children. Which of the following is the most common complication of this procedure?
A) Early implant rupture
B) Hematoma
C) Infection
D) Lifetime need for reoperation
E) Seroma
The correct response is Option D.
Augmentation mammaplasty is known to have high rates of complications including reoperation. Infection, seroma, hematoma, and early implant rupture are rare in elective, cosmetic augmentation mammaplasty.
A 49-year-old woman is evaluated because of a traumatic laceration of the right lower eyelid and cheek. Physical examination shows difficulty with eyelid closure, voluntary squinting, and animation. Which of the following branches of the facial nerve is most likely injured?
A) Buccal
B) Cervical
C) Marginal mandibular
D) Temporal
E) Zygomatic
The correct response is Option E.
Anatomically, the orbicularis oculi muscle is divided into three segments: pretarsal, preseptal, and orbital. However, functionally, the orbicularis oculi muscle is divided into the medial inner canthal orbicularis and the extracanthal orbicularis. The medial inner canthal orbicularis is responsible for blinking, lower lid tone, and the pumping mechanism of the lacrimal system. Innervation to the inner canthal orbicularis is from the buccal branches of the facial nerve. The zygomatic branch of the facial nerve innervates the extracanthal orbicularis, which controls eyelid closure, voluntary squinting, and animation. The temporal, marginal mandibular, and cervical branches do not provide innervation to the orbicularis oculi muscle.
A 28-year-old woman comes for follow-up evaluation 2 weeks after undergoing bilateral augmentation mammaplasty with subpectoral placement of 325-mL, round, smooth saline prostheses. She is now concerned that both prostheses appear “too high.” Physical examination shows fullness in the upper quadrants of both breasts. Which of the following interventions is most appropriate?
A)Administration of oral zafirlukast
B)Application of a circumferential breast band
C)Injection of corticosteroid into the inframammary crease
D)Open capsulotomy
E)Percutaneous release of the inframammary crease
The correct response is Option B.
The most appropriate recommendation is breast band application. Breast shape following augmentation mammaplasty undergoes dynamic changes. The skin envelope and pectoralis muscle stretch under the expansion pressure of the prosthesis. The skin of the lower pole will stretch, allowing the prostheses to migrate inferiorly. Breast massage and a circumferential elastic breast band applied around the superior breast encourage this migration.
Zafirlukast is a leukotriene-antagonist that is used for the treatment of asthma. Preliminary studies suggest improvement in capsule contractures. This drug is associated with potential life-threatening liver complications as well as neuropsychiatric events. Because administration in the scenario described would constitute an off-label use of the drug, extensive discussion with the patient would be required prior to use.
In the past, steroid was injected into the saline compartment of a double-lumen prosthesis in an attempt to decrease the incidence of capsule contraction. This delivery system was uncontrolled and many prostheses migrated beyond the normal limits of the inframammary crease. Postoperative steroid injection has been used with some success for the prevention of recurrent capsule contracture following capsulectomy.
If residual inferior pectoralis muscle fibers are left intact along the rib or capsule contracture develops, open capsulotomy may be required; however, conservative treatment is indicated at this early postoperative period.
Percutaneous release would expose the patient to unnecessary complications of prosthesis injury, bleeding, and inframammary crease malposition.
A 23-year-old woman with micromastia and bilaterally tuberous breasts comes to the office to discuss augmentation mammaplasty and improving the overall appearance of her breasts. Physical examination shows bilateral mildly ptotic breasts with glandular tissue herniating through the nipple-areola complex. The lower pole appears mildly deficient in the lower medial and lateral quadrants, and the distance from the nipple to the inframammary crease is 5.5 cm on stretch. Which of the following is the most appropriate operative approach for this patient?
A) Implant placement with circumareolar mastopexy
B) Implant placement without mastopexy
C) Implant placement with vertical mastopexy
D) Implant placement with Wise-pattern mastopexy
E) Two-stage reconstruction with tissue expander, followed by placement of a permanent implant
The correct response is Option A.
The tuberous breast deformity was first described by Aston and Rees in 1976. While most of the surgical approaches listed, with the exception of implants together with Wise-pattern mastopexy, have been described for the spectrum of tuberous breast deformities, the key is to select the right procedure for the right patient. In this case, a mild form of the deformity is described. Implant placement alone, even with parenchymal scoring and lowering of the inframammary crease, is unlikely to correct the deformity of the nipple-areola complex. In cases of severe ptosis, vertical mastopexy may be used but would be unnecessary in this patient with mild ptosis. In severely deficient cases, a two-stage approach with tissue expansion may be necessary, but it would be over-operating in this mildly deficient patient. Recently, fat grafting has also been advocated for this procedure.
In the case described, which is a common presentation, a periareolar approach is typically used to place the implant in a dual-plane configuration. Subglandular placement is also described. The inframammary crease is commonly adjusted downward. Radial scoring of the parenchyma and a circumareolar mastopexy are typically performed.
In the recent review by Kolker and Collins, 92% of tuberous breast patients had a one-stage procedure. Ninety-six percent of these were treated with implant placement and circumareolar mastopexy, combined with inframammary crease adjustment and radial scoring of the parenchyma.
A 32-year-old woman comes to the office for consultation regarding augmentation mammaplasty. She is concerned about the potential complications with the use of silicone gel prostheses within the first 5 years postoperatively. Which of the following is the most commonly reported complication of the implantation of cohesive silicone gel breast prostheses?
A)Capsular contracture
B)Granuloma
C)Hematoma
D)Infection
E)Rupture
The correct response is Option A.
Cohesive silicone gel is a breast prosthesis option that has been approved by the FDA since 2006. Cohesive gel prostheses have also been called ?gummy bear? prostheses. They maintain their shape because of the increased cross-linking within the silicone gel.
A study by Cunningham followed 1008 patients and 1898 cohesive gel prostheses. Rupture rate was 1.1% for aesthetics and 3.8% for reconstructive procedures. Capsular contracture rates (Baker III/IV) were 9.8/13.7%, and infection was 1.6/6.1%, respectively. Thus, capsular contracture was the most common of the listed complications. The reported incidence of hematoma is approximately 2%.
It should be noted that complications occur more commonly in primary reconstruction as compared to primary augmentation. These findings are important in the preoperative counseling of patients.
A 28-year-old woman comes to the office to discuss augmentation mammaplasty. She is interested in silicone implants, specifically highly cohesive gel shaped implants. Which of the following is the most likely result of increasing the cross-linking in these implants?
A) Decreased risk of gel fracture
B) Decreased risk of shell delamination
C) Improved form stability
D) Increased risk of folds
E) Softer implants
The correct response is Option C.
Increasing the cross-linking in a highly cohesive gel shaped silicone implant improves form stability. This allows for the creation of shaped implant designs that persist despite position or external forces on the implant.
The current, fifth-generation silicone breast implants derive their cohesiveness from the cross-linking of the silicone. Increasing the amount of cross-linking leads to an increase in cohesiveness and a firmer implant. This may lead to less rippling and folding because of resistance to collapse; however, recent MRI studies have shown folds and distortions are still possible. Increasing cohesiveness, however, does have some disadvantages with potential risks for gel fracture and delamination of the implant shell.
A 35-year-old woman, gravida 2, para 2, seeks implant-based augmentation mammaplasty. She breastfed both her children. Which of the following is the most common complication of this procedure?
A) Early implant rupture
B) Hematoma
C) Infection
D) Lifetime need for reoperation
E) Seroma
The correct response is Option D.
Augmentation mammaplasty is known to have high rates of complications including reoperation. Infection, seroma, hematoma, and early implant rupture are rare in elective, cosmetic augmentation mammaplasty.
A 37-year-old woman with macrotextured saline implants placed 10 years ago was recently diagnosed with a peri-implant fluid collection. Ultrasound-guided aspiration of the fluid is performed and sent for Wright-Giemsa–stained smears, and cell block immunohistochemistry/flow cytometry testing. Images are shown. Which of the following results would confirm a diagnosis of breast-implant–associated anaplastic large cell lymphoma?
A) CD30 negative, ALK negative
B) CD30 negative, ALK positive
C) CD30 positive, ALK negative
D) CD30 positive, ALK positive
The correct response is Option C.
This patient has breast implant–associated anaplastic large cell lymphoma (BIA-ALCL) which is a distinct form of CD30-positive T-cell, non-Hodgkin’s lymphoma that arises in association with a breast implant after either reconstructive or cosmetic surgery. The disease is typically contained within the capsule and fluid immediately adjacent to the implant. Patients commonly present with delayed seroma, but can also present with pain, capsular contracture, and/or a palpable mass. BIA-ALCL risk is higher with textured devices and these concerns led to the 2019 FDA recall of Allergan Biocell devices. The recall includes older McGhan and Inamed implants and current Natrelle implants. When patients with suspicious history or symptoms are evaluated, pre-operative imaging (e.g., mammography, ultrasound, and/or MRI) is recommended with aspiration of identifiable fluid or biopsy of mass. Ultrasound is considered the diagnostic modality of choice. Diagnostic evaluation should indicate concern for BIA-ALCL to the pathologist to include cytological evaluation of seroma fluid or mass with Wright-Giemsa stained smears and cell block immunohistochemistry/flow cytometry testing for cluster of differentiation (CD30) and anaplastic lymphoma kinase (ALK) markers. Wright-Giemsa staining shows pleomorphic cells with horseshoe shaped nuclei, nuclear folding and abundant vacuolated cytoplasm. All known cases of BIA-ALCL are CD30-positive and -negative for ALK, distinct from systemic ALCL which is ALK-positive.
A 47-year-old woman, gravida 3, para 3, is evaluated for improvement of breast appearance. She breast-fed all three of her children for 1 year each. Examination shows the distance from nipple to sternal notch is 27 cm bilaterally; decreased superior pole volume, and striae are also noted. There is Grade 3 ptosis bilaterally. The pinch of the superior pole soft tissue is 1 cm. Which of the following procedures is most likely to improve superior pole volume and breast shape in this patient?
A) Dual-plane implant augmentation
B) Mastopexy with dual-plane implant augmentation
C) Mastopexy with subglandular implant augmentation
D) Subglandular implant augmentation
E) Vertical mastopexy
The correct response is Option B.
Goals of improvement would be upper pole fullness and a coned, rounded breast, with raising the nipple. Because the superior pole thickness is less than 2 cm, a subglandular implant is not recommended. A dual-plane implant would not address the ptosis and would likely leave persistent ptosis. Vertical mastopexy alone would require some modification to address the excess vertical skin with some element of horizontal inferior excision. This would not address the lack of upper pole volume in the long term. The striae indicate poor tissue strength. Staged implant placement would have the fewest risks.
A 45-year-old woman comes for evaluation 1 year after undergoing vertical mastopexy without placement of prostheses because she thinks her breasts have started to sag. An increase in which of the following breast dimensions has most likely occurred since the patient’s last visit?
A) Breast base diameter
B) Nipple to inframammary crease
C) Nipple-areola diameter
D) Suprasternal notch to inframammary crease
E) Suprasternal notch to nipple
The correct response is Option B.
The nipple-to-inframammary crease dimension is most likely to increase over time. This leads to pseudoptosis (bottoming out) and the appearance of a sagging breast. Pseudoptosis occurs when the breast gland migrates lower than the inframammary crease while the nipple stays in normal position. It is essential that patients be informed that their breasts will eventually sag following mastopexy. Procedures to prevent this from occurring include the use of permanent mesh encircling the breast mound. Mastopexy and reduction mammaplasty share similar operative strategies as well as complications. All techniques suffer bottoming out to different degrees.
Breast base diameter will change very little over time as long as the breast volume remains constant; eg, weight gain can increase breast volume.
An increase in the nipple-areola diameter is unlikely with vertical mastopexy; however, increased areola diameter is associated with periareolar mastopexy. To minimize this complication, a permanent purse-string suture is recommended. Suprasternal notch-to-inframammary crease distance changes very little in comparison with the nipple-to-inframammary crease distance.
The suprasternal notch-to-nipple distance changes very little postoperatively. When a prosthesis is used during mastopexy, this distance will increase; however, the nipple-to-inframammary crease will usually increase to a greater extent.
In women undergoing augmentation mammaplasty with saline-filled implants, which of the following techniques is most likely to decrease the longevity of the implant and lead to early rupture?
(A) Underfilling of the implants below the manufacturer’s recommended minimum
(B) Filling of the implants to the manufacturer’s recommended minimum
(C) Filling of the implants to the volume between the manufacturer’s recommended minimum and maximum (D) Filling of the implants to the manufacturer’s recommended maximum
(E) Overfilling of the implants above the manufacturer’s recommended maximum
The correct response is Option A.
Adequate fill volume is recommended to increase the longevity of a breast implant. This decreases both fold-flaw failure and the potential for premature failure resulting from filling the implant at or below the manufacturer’s minimum volume. Implants are softer and more sloping at lower levels of fill volume, but these “underfilled” implants have also been shown to wrinkle more, leading to a shorter lifespan because of stresses induced by wrinkling. As the volume of the implant increases, palpable and visible wrinkling decreases, resulting in increased longevity.
Some studies have also shown that implant longevity can be maximized by filling the implants to the volume at which they exhibit the least wrinkles, even if it exceeds the manufacturer’s maximum recommended volume. However, this technique, known as “overfill,” is not recommended.
A healthy 24-year-old woman undergoes bilateral cosmetic breast augmentation with subglandular saline implants. Which of the following percentages best represents this patient’s 10-year risk for reoperation because of an implant-related indication?
(A) 5%
(B) 25%
(C) 50%
(D) 75%
(E) 95%
The correct response is Option B.
After breast augmentation with saline implants, the 10-year risk for reoperation for any implant-related indication is about 25%. Implant-related indications include deflation of the implant, capsular contracture, hematoma, wound infection, and seroma.
In one multicenter retrospective study of 450 patients with a mean follow-up period of 13 years, the reoperation rate for implant-related indications was 25.8%. In another multicenter retrospective study of 504 patients with a mean follow-up period of 6 years, the rate was 21%. In a third retrospective study of 749 women with a mean follow-up period of 5 years, the rate was 12% for cosmetic breast augmentations and 34% for breast reconstructions.
A 40-year-old woman comes to the office for consultation on an augmentation mastopexy 2 years after giving birth to her second child. She is back to her pre-pregnancy weight. Physical examination shows involutional changes contributing to a deflated appearance of the breasts. This appearance is most likely due to a histologic decrease in which of the following?
A) Area composed of stromal matrix
B) Number of differentiated lobules
C) Thickness of dermis
D) Thickness of pectoralis muscle
E) Volume of adipose tissue
The correct response is Option B.
Postpartum involutional changes can manifest clinically as breasts that appear deflated, commonly due to a loss of volume and skin that has been stretched. On a histologic level, these clinical manifestations occur due to a decrease in the number and area of differentiated lobules that were enlarged and specialized for milk production. As this occurs, it is hypothesized that the lobular area is then replaced by stromal matrix and eventually fat. Involutional changes do not refer to changes in the dermis, pectoralis muscle or chest wall structures.
A 28-year-old woman, gravida 2, para 2, undergoes augmentation mammaplasty 1 year post partum. On postoperative day 3, the patient comes to the office because of impaired wound healing at the incision site. Physical examination shows white viscous discharge leaking from the edge of the wound consistent with galactorrhea. Which of the following is the most appropriate management?
A) Administration of bromocriptine
B) Administration of metoclopramide
C) Administration of trimethoprim-sulfamethoxazole
D) Application of negative pressure wound therapy
E) Debridement of the wound edges with wet-to-dry dressings
The correct response is Option A.
There are incidents of surgical procedures of the breast associated with galactorrhea leading to skin breakdown, nipple necrosis, and cellulitis. A dopamine agonist such as bromocriptine will cause decreased lactation in cases of galactorrhea/galactocele, thereby improving wound healing. Antibiotics such as sulfamethoxazole and trimethoprim (Bactrim) are generally not required, because the exudate is sterile. There is no need for debridement of the wound edges. Negative pressure wound therapy may increase lactation and galactorrhea, further impairing wound healing. Metoclopramide is a dopamine antagonist used for nausea and vomiting.
Which of the following findings is most likely in a patient with Poland syndrome?
(A) Anomalies of the feet
(B) Bilateral abnormalities of the ribs
(C) Breast hypertrophy
(D) Hypoplasia of the pectoralis major muscle
(E) Polythelia
The correct response is Option D.
Poland syndrome is a congenital anomaly that is characterized by unilateral aplasia or hypoplasia of the pectoralis major muscle and adjacent musculoskeletal components. Chest wall anomalies can also be unilateral and include aplasia or hypoplasia of the breast or nipple, partial agenesis of the ribs and sternum, and anomalies of the shoulder girdle. Ipsilateral hand anomalies are common. In severe forms of the disease, the pectoralis, latissimus, and serratus muscles are completely absent.
Poland syndrome typically occurs sporadically and its etiology is not fully understood. Men and women are affected equally. Despite the absence of structures of the chest wall, patients have minimal physical disability. Appropriate reconstructive options include transfer of the latissimus in men and women, with the addition of submuscular augmentation mammaplasty in women.
Which of the following findings are consistent with tuberous breast syndrome?
(A) Deficiency of the skin envelope, a decrease in vertical breast height, breast hypoplasia, and absence of the pectoralis major muscle
(B) Deficiency of the skin envelope, a decrease in vertical breast height, breast hypoplasia, and areolar hypertrophy
(C) Deficiency of the skin envelope, elongation of vertical breast height, breast hypertrophy, and absence of the pectoralis major muscle
(D) Redundancy of the skin envelope, a decrease in vertical breast height, breast hyperplasia, and absence of the pectoralis major muscle
(E) Redundancy of the skin envelope, elongation of vertical breast height, breast hyperplasia, and areolar hypertrophy
The correct response is Option B.
Tuberous breast syndrome, also referred to as tubular breast syndrome or constricted breast syndrome, is comprised of a broad spectrum of features. Patients with tuberous breast syndrome may have any or all of several findings. These can include a deficiency in the skin envelope that can involve only one quadrant or can lead to severe constriction, a decrease in the overall vertical height from the top of the breast to the inframammary fold, hypertrophy of the areola, which is believed to compensate for constriction at the base of the breast, and a true deficit of breast tissue, particularly at the area of skin deficiency. After the skin is released surgically, volume must be added to create a normal-appearing breast.
Absence of the pectoralis major muscle is a feature of Poland’s syndrome.
A 31-year-old woman, gravida 2, para 2, presents for augmentation mammaplasty. She wants improved overall fullness with limited scarring. She currently wears a size 34A brassiere. Physical examination shows pseudoptosis. Submuscular augmentation with which of the following techniques is most appropriate to meet this patient’s goals?
A) Pectoralis muscle not released along inframammary fold
B) Release of pectoralis muscle along inframammary fold and periareolar mastopexy
C) Release of pectoralis muscle along inframammary fold and submammary dissection to inferior areola
D) Release of pectoralis muscle along inframammary fold and vertical mastopexy
E) Release of pectoralis muscle along inframammary fold only
The correct response is Option C.
The patient would benefit most from a type II dual-plane augmentation mammaplasty. Her breast parenchyma is moderately mobile over the pectoralis muscle, and she has moderate stretch of the lower pole skin. The goal of dual-plane augmentation is to maximize muscle coverage while allowing optimal lower pole expansion. It also allows redistribution of the breast tissue overlying a submuscular implant.
Soft tissue coverage of the implant is an important consideration regarding pocket location. If pinch thickness of the upper breast is 2 cm or greater, an implant can be placed above the pectoralis muscle. If pinch thickness is less than 2 cm, then the implant should be placed at least partially under the pectoralis muscle. If pinch thickness is less than 0.5 cm along the inframammary fold, then the pectoralis muscle should not be released along the inframammary fold.
There are three types of dual-plane augmentation. Type I releases the pectoralis muscle along the inframammary fold. This is used for most routine augmentation mammaplasties, with all the breast parenchyma above the inframammary fold, tight attachments of the parenchyma-muscle interface, and areola-to-inframammary fold stretch of 4 to 6 cm.
Type II dual-plane augmentation releases the pectoralis muscle along the inframammary fold and dissection is performed superficial to the pectoralis muscle to the inferior border of the areola. This is used for augmentation mammaplasties with most of the breast parenchyma above theinframammary fold, looser attachments of the parenchyma-muscle interface, and areola-to-inframammary fold stretch of 5.5 to 6.5 cm.
Type III dual-plane augmentation releases the pectoralis muscle along the inframammary fold, and dissection is performed superficial to the pectoralis muscle to the superior border of the areola. This is used for augmentation mammaplasties in patients with glandular ptosis or true ptosis, when a third of the breast parenchyma is below the inframammary fold, there are very loose attachments of the parenchyma-muscle interface, and areola-to-inframammary fold stretch is 7 to 8 cm. Type III can also be used in breasts with constricted lower poles.
Mastopexy increases scarring, which this patient wanted to limit, and is often not needed with the appropriate dual-plane approach.
A 28-year-old woman is scheduled to undergo release of severe cicatricial contraction six months after removal of an infected breast prosthesis. Preoperative physical examination shows that soft tissue is required in the inframammary area. Closure with a submammary flap is planned. Which of the following vessels is most likely to supply blood to this flap?
(A) Internal mammary perforators
(B) Lateral thoracic artery
(C) Superficial inferior epigastric artery
(D) Thoracoacromial perforators
(E) Thoracodorsal perforators
The correct response is Option A.
The sequelae of infection in breast augmentation can be severe cicatricial contraction of the inferior pole of the breast. Reoperation can be considered after an appropriate interval of six months, which allows for resolution of inflammation and scar maturation. If additional soft tissue is required, submammary flaps (with good color and texture match) can be used from the medial or lateral base of the breast. The blood supply is based medially on perforators of the internal mammary or superior epigastric arteries and laterally from perforators of the intercostal vessels.
The lateral thoracic artery is the blood supply for a lateral chest flap, which would not be able to contribute any meaningful amount of soft tissue to the submammary area.
The superficial inferior epigastric artery is the basis of lower abdominal flaps and has no role as a local flap for breast surgery.
The thoracoacromial perforators are associated with extended cheek/neck flaps for head and neck reconstruction.
Thoracodorsal perforators are the basis of thoracodorsal perforator flaps and would not be able to contribute soft tissue to the submammary area.
A 35-year-old woman comes to the office for consultation because she is dissatisfied with the appearance of her “deflated” and “saggy” breasts. Augmentation/mastopexy is planned. Compared with placement of the implant in the subglandular position, placement of the implant in the subpectoral space will preserve blood supply to the breast tissue and skin through which of the following arteries?
A) Internal thoracic
B) Lateral thoracic
C) Superficial superior epigastric
D) Thoracoacromial
E) Thoracodorsal
The correct response is Option D.
The perfusion of the nipple-areola complex is a major concern during breast procedures involving periareolar and intraparenchymal incisions. The nipple-areola complex has a very rich and overlapping perfusion through multiple sources. This fact allows the design of various pedicles to carry the nipple and areola with different techniques. The blood supply through the internal thoracic vessels reaches the breast, nipple, and areola through the intercostal perforators, which may be divided during both subpectoral and subglandular implant placement.
The location of the implant deep or superficial to the pectoralis muscle will not change the perfusion through the superficial epigastric vessels. The same is true for the blood supply through the lateral thoracic vessels. However, the flow through the thoracoacromial vessels to the breast parenchyma will be preserved by placement of the implant deep to the pectoralis muscle. Creation of a subglandular pocket above the muscle will interrupt the collaterals from the thoracoacromial vessels through the muscle to the parenchyma.
The thoracodorsal artery is not a major source of blood supply to the breast and the position of the implant will not affect it.
A 47-year-old woman, gravida 2, para 2, who has grade III breast ptosis is evaluated for mastopexy. Attenuation of which of the following structures is the most likely cause of the ptosis?
A) Breast acini
B) Cooper ligaments
C) Lactiferous ducts
D) Scarpa’s fascia
E) Subdermal plexus
The correct response is Option B.
Breast ptosis is a complex interaction of events, informed by breast size, gravity, aging, lactation, and parity. It occurs through a combination of atrophy of the breast tissue, loss of elasticity of the skin envelope, and attenuation of Cooper ligaments.
While the breast is surrounded by fascia, the continuation of Scarpa’s fascia forms the posterior capsule of the breast.
The lactiferous ducts and breast acini do not contribute significantly to ptosis.
Subdermal plexus provides vascularity rather than support to the breast.
A 55-year-old woman who underwent augmentation mammaplasty with retro-pectoral smooth saline implants 18 years ago comes to the office because she is dissatisfied with her breast shape. Physical examination shows glandular ptosis hanging off the implants. She has Grade I capsules. Which of the following is the most appropriate procedure to correct this patient’s deformity?
A) Implant exchange alone
B) Implant exchange with capsulectomy
C) Implant exchange with mastopexy
D) Implant exchange with suture plication of the expanded inferior pocket
E) Site change to subglandular placement
The correct response is Option C.
The described patient has a “snoopy nose deformity” or “waterfall breast deformity,” with the ptotic breast hanging off of the implant. There is no pocket expansion. The implants have stayed in their original position while the native breast tissue has become ptotic with time and gravity. This is not superior malposition due to capsular contracture; both breasts are soft. Correction of this problem is best performed with an appropriately chosen form of mastopexy. In this case, replacement of the implants would also be performed because of their age.
Implant exchange alone would not correct the ptotic breast. Capsulectomy is not indicated, since the breasts are soft, and no capsule is noted clinically. Similarly, suture plication of the pocket is not required, since the inframammary fold is in the correct position, and no second fold is seen. Site change would not correct the patient’s grade III ptosis, but it may be used to correct this problem in cases without significant ptosis.
A 52-year-old woman with a 15-year history of subglandular breast implants presents with concerns about obtaining her first mammogram. She states that she has avoided mammography because she is concerned about implant rupture from the procedure. Which of the following is the most appropriate next step?
A) Diagnostic mammography with Eklund views
B) Diagnostic mammography with Waters views
C) MRI with contrast
D) Screening mammography with Eklund views
E) Screening mammography with Waters views
The correct response is Option D.
The correct answer is screening mammography with Eklund views. Some women report that they are hesitant to undergo screening mammography because they are afraid of an implant rupture. A study of breast implant complications reported to the Food and Drug Administration (FDA) demonstrated that of 714 breast implant adverse events reported, 66 described rupture or problems directly associated with mammography. In addition to implant rupture, an oncologic concern is inadequate x-ray views for assessment of the breast tissue.
Eklund described a modified position for mammography in which the breast tissue is displaced in front of the implant. This allows for adequate assessment of the breast tissue. Screening mammography is appropriate for routine mammography; diagnostic mammography is used to further characterize mammographic concerns or in the case of known pathology.
While MRI is used to detect silicone implant rupture and as an adjunct in oncological screening, it is not currently recommended as a routine primary screening tool.
It should be noted that the presence of breast implants should not be used to justify deferral of recommended oncologic screening, and the true incidence of breast implant rupture caused by mammography is difficult to assess given the different generations of implants and the possible presence of capsular contracture.
Waters views are used to assess the maxillary sinuses.
A 38-year-old woman comes to the office for consultation regarding surgical correction of sagging of the breasts. She breast-fed three children during the past five years; her youngest child was weaned two years ago. Physical examination shows second-degree ptosis. For this patient, which of the following is an advantage of mastopexy with augmentation over mastopexy alone?
(A) Decreased risk of loss of nipple sensation
(B) Decreased risk of nipple malposition over time
(C) Decreased stretch deformity of surgical scars
(D) Increased longevity of correction of ptosis
(E) Increased upper pole volume
The correct response is Option E.
The combination of implantation of a prosthesis with mastopexy can enhance the size and contour of the breast. This procedure often reduces the length of the incisions required to correct the ptosis because of the volume enhancement delivered by the implant.
There is no known difference in the degree of loss of sensation between the two methods. The weight of the prosthesis places additional tension at the site of incision, causing more rapid recurrence of ptosis. This is especially true for larger prostheses placed in the subglandular position. There is an increased risk of nipple malposition because the nipple is moved at the same time as the implant.
A 36 €‘year-old woman comes to the office for consultation regarding mastopexy. She will not consider use of prostheses and is concerned about the length of the scars. Photographs of the breasts are shown. Which of the following types of mastopexy is most appropriate for this patient?
(A) Circumareolar
(B) Crescent
(C) Vertical
(D) Wise €‘pattern
The correct response is Option C.
The most appropriate management for the patient described, who has grade 2 ptosis of the breasts, is a vertical mastopexy. The procedure will leave periareolar and vertical scars but will give the patient a longer-lasting result than a periareolar procedure.
Ptosis is graded on a scale of 1 to 3, depending on nipple position changes from above or below the level of the inframammary crease. In pseudoptosis, the nipple is above or at the level of the crease, but the majority of breast parenchyma has descended and is distributed below the inframammary fold. In grade 1 ptosis, the nipple position is within 1 cm of the level of the inframammary fold. In grade 2 (moderate ptosis), the nipple is clearly below the fold (1 to 3 cm) but above the lowest part of the breast. In grade 3 ptosis, the nipple is greater than 3 cm below the inframammary fold and below the lower contour of the gland.
In grade 1 ptosis, subglandular augmentation is often adequate. Alternatively, if the patient does not desire implants, a dermal or crescent mastopexy, which involves excision of a crescent-shaped area of skin above the areola, may be necessary. Circumareolar mastopexy, which involves concentric excision of skin and leaves no vertical scar beneath the areola, is also adequate for grade 1 ptosis. Periareolar resections without implant placement tend to flatten the shape of the breast.
A vertical or infraareolar mastopexy is ideal for grade 2 or moderate ptosis.
Wise €‘pattern mastopexy is appropriate for grade 3 ptosis with large amounts of skin excess, but the procedure will leave an inverted T €‘shaped scar.
Which of the following cell types is most associated with the chronic inflammation that leads to breast implant–associated anaplastic large cell lymphoma?
A) B-cells
B) Monocytes
C) Neutrophils
D) Red blood cells
E) T-cells
The correct response is Option E.
Evidence suggests that chronic inflammation is the stimulus responsible for the development of breast implant–associated anaplastic large cell lymphoma (ALCL) and T-cells are the predominant cell type responding to this antigenic stimulus. B-cells have been implicated in orthopedic implant lymphomas. The other cell types are involved in inflammation, but they are not associated with breast implant-associated ALCL.
A 21-year-old woman desires surgical correction because her left breast has an abnormal appearance. On examination, the diameter of the left breast is more narrow at the base than at the midportion, and there is superior displacement of the inframammary fold. The areola is disproportionally enlarged, and the breast tissue appears to be herniating into the areola. The left cup size of her bra is 32B, and the right cup size is 32C. The right breast is normal.
Which of the following is the most appropriate management?
(A) Right-sided vertical breast reduction
(B) Pedicled TRAM flap reconstruction of the left breast
(C) Wise-pattern breast reduction on the right with lowering of the inframammary fold
(D) Augmentation mammaplasty on the left using a saline-filled implant
(E) Augmentation mammaplasty on the left with radial scoring and areolar reduction
The correct response is Option E.
This patient has a tuberous, or constricted, breast deformity. Affected patients have unilateral narrowing of the breast; the breast tissue appears to be herniating into the areola. In order to adequately correct this deformity, implant augmentation mammaplasty should be combined with repositioning of the inframammary fold, radial scoring of the breast parenchyma, and reduction of the herniated tissue and areola. This will correct the size and shape discrepancies, resulting in a left breast that appears similar to the unaffected right breast.
The right breast should not be reduced by any method to match the size and shape of the abnormal left breast. TRAM flap reconstruction is associated with significantly higher morbidity and should not be performed as initial management. Implantation alone will enlarge the left breast but will not correct the abnormal shape of the breast.
A 24-year-old woman comes to the office for consultation regarding surgical correction of the breast deformity shown above. Which of the following is the most appropriate management?
(A) Augmentation with Wise-pattern mastopexy of both breasts
(B) Augmentation with periareolar mastopexy of both breasts
(C) Latissimus dorsi myocutaneous flap reconstruction of the left breast and periareolar mastopexy of the right breast
(D) Extended dorsi myocutaneous flap reconstruction of the left breast and periareolar reduction of the right breast
(E) Transaxillary augmentation of the left breast and periareolar mastopexy of the right breast
The correct response is Option B.
The patient described has tuberous breast deformity, which is characterized by three components: herniation of the breast tissue into the nipple-areola complex with a cylindrical projection accompanied by a relatively large areola; deficiency of the lower pole of the breast in both vertical and horizontal axes; and hypoplasia. Periareolar mastopexy with augmentation will give access for radial-releasing incisions, which will allow expansion of the base of the breast and simultaneous areolar reduction. Wise-pattern mastopexy is indicated for more severe breast ptosis. Reconstruction with latissimus flap is a form of treatment for congenital chest wall deformities such as Poland syndrome. Contralateral reduction will not address the tuberous deformity problem. Transaxillary augmentation of the breast will not correct the nipple-areola complex or the constricted base.
A 70-year-old woman considers a mastopexy to address the ptosis she has
experienced after the onset of menopause. A decrease in which of the following
hormones is most responsible for the involutional changes of the breast glandular
parenchyma?
A ) Estrogen
B ) Follicle-stimulating hormone
C ) Luteinizing hormone
D ) Prolactin
E ) Testosterone
The correct response is Option A.
The hormone decrease most responsible for the involutional changes of the breast glandular
parenchyma during menopause is estrogen. During menopause, estrogen concentrations
decrease, promoting glandular involution with decreased cellularity and vascularity of the
glandular tissue. Prolactin decreases are less associated with menopausal changes to the
breast. Luteinizing hormone and follicle-stimulating hormone have important impacts during
different phases of the menstrual cycle before menopause and may both increase in
concentration during menopause. Testosterone changes do not primarily impact the breast
parenchyma during menopause.
REFERENCES:
1. Jones GE. Part I: basic principles: 1 anatomy, physiology, and aesthetics. In: Jones G, ed.
Bostwick’s Plastic and Reconstructive Breast Surgery. 4th Ed. Thieme. New York; 2020.
doi:10.1055/b-006-163726
2. Jatoi I, Benson J, Sbitany H. Anatomy. In: Atlas of Breast Surgery. Springer, Cham;
A 37-year-old woman comes to the clinic to be evaluated for augmentation mammaplasty to improve her breast shape. She is gravida 3, para 3, and breast-fed all of her children. On examination, she has decreased superior pole volume, and the distance from nipple to sternal notch is 28 cm. The nipple-areola complex is below the inframammary fold by 4 cm and is at the lower contour of the breast. Which of the following Regnault classifications of ptosis best describes these findings?
A) Grade I
B) Grade II
C) Grade III
D) Pseudoptosis
The correct response is Option C.
The Regnault classification of breast ptosis is based on the relationship of the nipple to the inframammary fold (IMF) and to the lower contour of the gland.
Pseudoptosis is the not true ptosis. In this situation, the nipple is above the level of the IMF but the breast parenchyma has descended below the IMF.
Grade I is minor ptosis with the nipple at the level of the IMF and above the lower contour of the gland.
Grade II is moderate ptosis with the nipple below the level of the IMF and above the lower contour of the gland.
Grade III is major ptosis with the nipple below the level of the IMF and at the lower contour of the gland.
A 24-year-old woman comes to the office one year after undergoing secondary augmentation mammaplasty because she reports that with manipulation she €œcan move each breast to the other side. € Physical examination shows that each breast prosthesis can be moved across the chest midline. Which of the following is the LEAST likely cause of this finding?
A ) Multiple procedures
B ) Preexisting chest wall deformity
C ) Prostheses with large base diameter
D ) Saline prostheses
E ) Subpectoral positioning
The correct response is Option D.
Synmastia is defined as any situation in which the breast prosthesis crosses the midline, even if it is only on one side. This relatively rare complication is at some times obvious and at other times more subtle, requiring breast manipulation to become apparent. There is no correlation with the use of either silicone- or saline-filled prostheses. The complication is more common in cases in which large prostheses with large base diameters are used, in multiple successive enlargement procedures, when there is a preexisting chest wall deformity, and with the subpectoral positioning of prostheses.
In a patient with breast implants, each of the following has been shown to interfere with screening mammography EXCEPT
(A) Baker III capsular contracture
(B) implant location
(C) implant size
(D) native breast volume
The correct response is Option C.
Several factors have been shown to affect the findings on mammography in women with breast implants. The positioning of the implant and the degree of associated capsular contracture have been known to influence the quantity of breast tissue that can be visualized. In addition, one study showed an increase in the amount of tissue that can be visualized postoperatively in a subset of women with small native breast volume. Therefore, it is important for patients who have breast implants to undergo mammographic evaluation at specialized centers experienced at obtaining mammograms using either compression or displacement (Eklund) techniques, which maximize visualization of the breast parenchyma.
The size of the implant has not been shown to affect the amount of breast tissue that can be visualized on mammography.
A 25-year-old woman is considering augmentation mammaplasty with silicone prostheses. The patient asks about the associated risks of developing connective tissue disease. Which of the following risk assessments is most accurate in this patient?
A)Increased risk of extracapsular leak only
B)Increased risk of intra- and extracapsular leak
C)Increased risk only if the silicone migrates to the lymph node
D)Increased risk only in the pre-1990 prostheses
E)No increased risk
The correct response is Option E.
Concern regarding an association between silicone breast prostheses and connective tissue disease was raised in the 1980s and early 1990s, eventually leading to the US Food and Drug Administration (FDA) moratorium of the use of silicone breast prostheses in augmentation mammaplasty. Since then, multiple cohort studies and case control studies in Europe and North America have failed to determine a causative association between silicone breast prostheses and any traditional or atypical connective tissue diseases.
A 28-year-old postpartum woman comes to the office for evaluation of breast asymmetry with pain and enlargement of the right breast for 2 months. Medical history includes augmentation mammaplasty 4 years ago. She denies fever or chills. She was previously breast-feeding but stopped this 1 month ago. Physical examination shows the right breast is significantly larger than the left breast. A well-healed peri-areolar incision is present and no evidence of infection is noted. Ultrasound shows a complex cyst, which yields 150 cc of milky fluid. A drain is placed. The most appropriate next step is administration of which of the following medications?
A) Bromocriptine
B) Cephalexin
C) Fluconazole
D) Prolactin
E) Trimethoprim-sulfamethoxazole
The correct response is Option A.
This postpartum patient is presenting with a symptomatic galactocele after breast-feeding. Galactoceles are benign breast cysts containing milk. They typically occur in women of childbearing age in the setting of active lactation, recent pregnancy, or the use of hormonal medications such as oral contraceptives. The galactocele is thought to occur from ductal obstruction. Although the presence of a breast implant and the respective pocket placement is unknown to have an effect on the development of galactoceles, there is some thought that peri-areolar incisions may contribute to the ductal obstruction. There are, however, documented cases of post-augmentation galactocele without peri-areolar incisions.
Treatment for a galactocele is typically medical with the initiation of oral bromocriptine. Bromocriptine is a dopamine receptor agonist and causes inhibition of prolactin secretion, which is the primary hormone responsible for milk production. Dosage is titrated to effect. Incision and drainage of the cyst, particularly in the setting of implants, is often performed as well to rule out the possibility of infection.
Cephalexin and trimethoprim-sulfamethoxazole are antibiotics and are not indicated in this case because there is no active infection. Fluconazole is indicated for the treatment of fungal infections. Prolactin would actually stimulate milk production and would worsen the patient’s symptoms.
In implantation of saline breast prostheses, which of the following fill levels is most likely to result in rupture due to fold flaw?
(A) Above the manufacturer’s recommended maximum
(B) Below the manufacturer’s recommended minimum
(C) Between the manufacturer’s recommended minimum and maximum
(D) Manufacturer’s recommended maximum
(E) Manufacturer’s recommended minimum
The correct response is Option B.
Based on engineering principles, studies have shown that implant longevity requires an adequate fill level to decrease fold-flaw failures and premature failures that result from underfilling (filling at or below the manufacturer’s recommended minimum level). They have also shown that filling the implants to their least-wrinkled fill level increases implant longevity and decreases premature failure. This generally requires overfilling exceeding the manufacturer’s recommended maximum level.
At lower fill levels, implants are softer and more sloping in contour but tend to wrinkle more and have a demonstrably shorter life span because of stress caused by wrinkling. As implant volume increases, palpable and visible wrinkling decreases and longevity increases.
A 50-year-old woman comes to the office for consultation about improving the appearance of her “saggy” breasts. She has lost 100 lb (45 kg) during the past 18 months by diet. Photographs are shown. Physical examination shows breast deflation and marked ptosis. A Wise pattern mastopexy with augmentation mammaplasty is planned. Which of the following arteries is most likely to provide circulation to the breast gland and nipple during submuscular augmentation in this patient?
A) Intercostal
B) Pectoral
C) Superior epigastric
D) Thoracoacromial
E) Thoracodorsal
Correct answer is option D.
The thoracoacromial artery and vein travel just deep to the pectoralis major muscle, supplying circulation to the overlying breast tissue and skin. Subglandular augmentation mammaplasty disrupts the connection between the thoracoacromial vessels and the overlying breast. This leads to a higher risk of wound-healing complications when placing the prosthesis in the subglandular plane. The submuscular plane of dissection maintains the connection between the thoracoacromial vessel and overlying breast and skin, allowing better potential healing. Intercostal arteries are multiple and are not completely disconnected with either subglandular or subpectoral augmentation mammaplasty. The superior epigastric artery provides circulation to the rectus abdominis muscle and abdomen. This artery would be injured with the mastopexy procedure. The thoracodorsal artery supplies the latissimus dorsi muscle and not the chest.
In grade II ptosis of the breast, which of the following best describes the position of the nipple?
A) At the apex of the breast mound
B) At the lowest contour of the breast
C) At the transposed inframammary fold
D) Between the inframammary fold and the lowest contour of the breast
E) On the posterior aspect of the breast as it rests on the chest wall
The correct response is Option D.
The classic Regnault definition of breast ptosis classifications are as follows:
Grade I: Nipple at the level of the inframammary fold
Grade II: Nipple between the level of the inframammary fold and the lowest contour of the breast
Grade III: Nipple at the lowest contour of the breast
A 40-year-old woman comes to the office because of firmness of the right breast. Twenty years ago, she underwent augmentation mammaplasty with smooth silicone prostheses placed in subglandular pockets. Which of the following is the most appropriate management?
A)Injection of corticosteroids
B)Treatment with zafirlukast (Accolate)
C)Closed capsulotomy
D)Open capsulotomy
E)Total capsulectomy
The correct response is Option E.
In the patient described with a capsular contracture, the most appropriate option is open capsulectomy. As opposed to open capsulotomy, open capsulectomy removes the entire capsule. Leaving the capsule behind in open capsulotomy can contribute to late seromas. Scar tissue left behind during an open capsulotomy may also prevent the prosthesis and breast from obtaining a natural shape.
Closed capsulotomy is no longer advised for breast prostheses because of the risk of rupturing the prosthesis during the procedure. Open capsulotomy and open capsulectomy with replacement of the prosthesis in the subglandular plane will continue to be associated with higher capsular contracture rates than submuscular or dual-plane placement. These are options for the patient as long as she understands the trade-offs of keeping the prosthesis in this plane.
Zafirlukast (Accolate) is a leukotriene receptor antagonist that is used as a bronchodilator in the management of asthma. The evidence supporting its use in capsular contracture is anecdotal. It is not approved by the US Food and Drug Administration (FDA) for use in capsular contracture; therefore, its use in the scenario described would be considered an ?off-label? indication. As such, zafirlukast cannot be recommended for the routine treatment of capsular contracture.
A 22-year-old woman who is dissatisfied with the appearance of her breast seeks surgical correction. Examination shows a constricted base width, widened areola, and areola herniation. Which of the following is the most likely diagnosis?
A) Athelia
B) Hypomastia
C) Poland syndrome
D) Pubertal arrest
E) Tuberous breast
The correct response is Option E.
The examination findings are consistent with the diagnosis of tuberous breast. Athelia is defined as absence of the nipple. Hypomastia would include a component of small breast volume, not described in this scenario. Poland syndrome often presents with an underdeveloped chest wall, including potential absence of the pectoralis major muscle. Pubertal arrest might allow for a disproportionately widened areola and apparent herniation if the breast bud had not fully developed, but it would not likely be associated with a constricted breast base width, which demonstrates breast development past the early stages of sexual development (known as Tanner stages).
A French woman, who underwent placement of Poly Implant Prothèse (PIP) gel implants in 2009, comes to the office for consultation because she had heard that the implants were filled with a nonmedical grade silicone. She reports that she has not had any problems with the implants, but would like to know the implications of retaining the implants and whether she should have them removed. This patient should be told that she is at increased risk for which of the following complications if she retains the implants?
A) Breast cancer
B) Cytotoxicity
C) Heavy metal poisoning
D) Implant rupture
E) Siloxane poisoning
The correct response is Option D.
The final report, in conjunction with the Department of Health in Australia, has shown a 2 to 6 times increased rupture rate in Poly Implant Prothèse (PIP) implants, which is detectable within 5 years of implantation. Increased levels of siloxane have been detected, but are not considered a health risk. No organic impurities have been detected and platinum levels are decreased in PIP gel compared with medical grade silicone. There is no increased breast cancer risk and no evidence of cytotoxicity. In the light of the increased rupture rate and the nonmedical grade nature of PIP silicone gel, the following recommendations were made:
all providers of breast implant surgery should contact any women who have or may have PIP implants, if they have not already done so, and offer them a specialist consultation and any appropriate investigation to determine if the implants are still intact;
if the original provider is unable or unwilling to do this, a woman should seek referral through her general practitioner to an appropriate specialist;
if there is any sign of rupture, she should be offered an explantation;
if the implants still appear to be intact, she should be offered the opportunity to discuss with her specialist the best way forward;
if, in the light of this advice a woman decides with her specialist that, in her individual circumstances, she wishes to have her implants removed, her health care provider should support her in carrying out this surgery. Where her original provider is unable or unwilling to help, the NHS will remove, but not normally replace, the implants;
if a woman decides not to seek early explantation, she should be offered annual follow up in line with the advice issued by the specialty surgical associations in January 2012. Women who make this choice should be encouraged to consult their doctor if they notice any signs of tenderness or pain, or swollen lymph glands in or around their breasts or armpits, which may indicate a rupture. At the first signs of rupture, they should be offered removal of the implants.
A 48-year-old woman comes to the office because she is dissatisfied with the “sagging” appearance of her breasts. Physical examination shows the location of the nipples 1 cm above the inframammary fold bilaterally. The majority of breast tissue is below the fold. Which of the following is the most likely diagnosis?
A ) Grade 1 ptosis
B ) Grade 2 ptosis
C ) Grade 3 ptosis
D ) Pseudoptosis
The correct response is Option D.
Regnault defined the degree of ptosis by evaluating the relationship of the nipple to the inframammary fold.
In pseudoptosis, the nipple is above or at the level of the inframammary fold, with the majority of the breast tissue below. This gives the impression of ptosis.
In Grade 1, or mild ptosis, the nipple is within 1 cm of the level of the inframammary fold and above the lower contour of the breast and skin envelopes. In Grade 2, or moderate ptosis, the nipple is 1 to 3 cm below the inframammary fold but above the lower contour of the breast and skin envelopes. In Grade 3, or severe ptosis, the nipple is more than 3 cm below the inframammary fold and below the lower contour of the breast and skin envelopes.
In women with breast ptosis, which of the following is an advantage of performing combined one-stage augmentation mammaplasty with mastopexy compared with mastopexy alone?
A) Better predictability of cosmetic outcome
B) Decreased complication rates
C) Decreased revision rates
D) Improved upper pole projection
E) Lower operative costs
The correct response is Option D.
Early reports have raised concerns about the safety of combined augmentation mammaplasty with mastopexy surgeries. However, in patients who wish to correct their breast ptosis these two procedures are often combined to a one-stage surgery and can show favorable outcomes. Nevertheless, plastic surgeons advocate that these cases should only be performed by experienced physicians.
Reasons are that the overall aesthetic results are harder to predict in one-stage augmentation/mastopexy procedures compared to mastopexy alone or even the two-stage augmentation mammaplasty followed by mastopexy. Both complication and revision rates are highest in the one-stage approach that combines augmentation mammaplasty with mastopexy. Longer operative time and the need for implants naturally increase operative costs.
The advantage of the simultaneous insertion of implants is the improved superior pole projection that cannot be achieved by mastopexy alone.
A 28-year-old woman is evaluated for micromastia. During consultation, she reports that her best friend underwent breast augmentation that was complicated by painful capsular contracture. Which of the following measures is most likely to prevent this complication in this patient?
A) Initiation of implant massage on postoperative day 5
B) Placement of a closed suction drain for prevention of postoperative hematoma
C) Use of a surgical support bra postoperatively for 2 weeks
D) Use of a subglandular, smooth, round implant via periareolar incision
E) Use of a subpectoral, textured implant via inframammary incision
The correct response is Option E.
Capsular contracture occurs when there is fibrosis of the peri-implant capsule. The severity is typically described by the Baker Grade classification.
Grade 1: the breast is soft and appears normal in size and shape
Grade 2: the breast is a little firm and appears normal
Grade 3: the breast is firm and appears abnormal
Grade 4: the breast is firm, appears abnormal, and is painful
Studies have shown a decreased relative risk for Baker grade 3-4 capsular contracture in primary breast augmentation associated with inframammary fold incision, textured implants, and subpectoral placement. The relative risk for capsular contracture was increased with periareolar or axillary incision, smooth implants, and subglandular placement. There is no evidence that wearing a support bra or implant massage will decrease the risk for capsular contracture. While hematoma is linked to capsular contracture, the presence of a drain does not prevent hematoma.
A 35-year-old woman comes for consultation regarding breast prosthesis removal because she is concerned about her risk of cancer. Specifically, she has read about anaplastic large cell lymphoma in women with breast prostheses. She underwent augmentation mammaplasty with saline breast prostheses 5 years ago. Physical examination shows absence of contracture and satisfactory position. Which of the following is the most appropriate next step in management?
A) Complete blood cell count
B) Evaluation by a hematologist
C) MRI of the breasts
D) Prosthesis removal
E) Reassurance
The correct response is Option E.
The US Food and Drug Administration (FDA) searched its adverse event reporting systems for reports received between January 1, 1995 and December 1, 2010, including information submitted by manufacturers as part of their required post-approval studies. This search identified 17 reports of possible anaplastic large cell lymphoma (ALCL) in women with breast prostheses. Although ALCL is extremely rare, the FDA believes that women with breast prostheses may have a very small but increased risk of developing this disease in the scar capsule adjacent to the prosthesis. Based on available information, it is not possible to confirm with statistical certainty that breast protheses cause ALCL. Currently, it is not possible to identify a type of prosthesis (silicone gel versus saline) or a reason for implantation (reconstruction versus aesthetic augmentation) associated with a smaller or greater risk.
When ALCL occurs, it has been most often identified in patients undergoing prosthesis revision procedures for late-onset, persistent seroma. Because it is so rare and most often identified in patients with late onset of symptoms such as pain, lumps, swelling, or asymmetry, it is unlikely that increased screening of asymptomatic patients would change their clinical outcomes. The FDA does not recommend prophylactic breast prosthesis removal in patients without symptoms or other abnormalities.
A patient with suspected ALCL should be referred to an appropriate specialist for evaluation. When testing for ALCL, fresh seroma fluid and representative portions of the capsule should be collected and sent for pathology tests to rule out ALCL. Diagnostic evaluation should include cytologic evaluation of seroma fluid with Wright-Giemsa–stained smears and cell block immunohistochemistry testing for cluster of differentiation and anaplastic lymphoma kinase markers. Any confirmed cases of ALCL in women with breast prostheses must be reported to the FDA.
Which of the following most appropriately describes the biomechanical characteristic specifically designed to minimize gel diffusion in a third-generation silicone prosthesis?
A ) Increased cross-linking of silicone elastomer
B ) Increased molecular weight of silicone gel
C ) Internal barrier coating
D ) Texturing of prosthesis surface
The correct response is Option C.
Several generations of silicone gel breast prostheses have been manufactured since the early 1960s. Third-generation prostheses manufactured since 1986, and recently FDA-approved for cosmetic and reconstructive procedures in the United States, were specifically developed to address the more common problems associated with second-generation prostheses, including silicone bleed. While a number of biomechanical properties were altered, the modification that was specifically designed to reduce silicone bleeding was the addition of an inner barrier on the elastomer shell. This barrier changed the solubility characteristics of the shell, thus inhibiting the diffusion of silicone through it. McGhan Medical released a prosthesis that had a diphenyl silicone copolymer barrier layer between an inner and outer layer of high-performance elastomer (Intrashiel). The Dow-Corning Silastic II prosthesis had a fluorosilicone copolymer layer to restrict silicone bleed.
The design of silicone gel breast prostheses has evolved significantly since their first introduction. First-generation prostheses had thick shell walls, viscous gel, and Dacron patches. Hardness and contracture were major complaints. To address these issues, second-generation prostheses were developed (1973-1985), which had thinner walls and lower viscosity gel. The result was a softer, more natural-feeling prosthesis in the early postoperative period. Second-generation prostheses had the highest rates of rupture, bleed, and capsular contracture, and as a result, third-generation prostheses reintroduced thicker shells and more cohesive gel.
Increasing the cross-linking of the silicone elastomer strengthens and thickens the wall of the prosthesis. Texturing of the prosthesis surface was a strategy designed to decrease the rate of capsular contracture.
A 33-year-old woman comes to the office for consultation because she is dissatisfied with the “sagging” appearance of her breasts. Examination shows grade II ptosis and loss of fullness in the upper pole. A vertical mastopexy is planned. The most common medial innervation to the nipple-areola complex is the anterior cutaneous branches of which of the following intercostal nerves?
A) Second and third
B) Third and fourth
C) Fourth and fifth
D) Fifth and sixth
E) Sixth and seventh
The correct response is Option B.
The most common medial innervation of the nipple-areola complex is mainly 57% from the anterior cutaneous branches of the third and fourth intercostal nerves. The third intercostal nerve accounts for 21.4%. They always reach the areolar edge between 8 and 11 o’clock on the left and 1 and 4 o’clock on the right. The nerve innervation to the nipple-areola complex is important in planning different incisions around the areola in both reduction mammaplasty and mastopexy.
A 32-year-old woman is scheduled to undergo augmentation mammaplasty with highly cohesive, anatomically shaped, silicone-filled breast implants. She asks the surgeon about postoperative monitoring for implant rupture. This patient should be counseled that, according to FDA recommendations, postoperative monitoring for rupture most appropriately includes which of the following?
A) Manual examination 3 years postoperatively, then annually thereafter
B) MRI screening 2 years postoperatively, then every 3 years thereafter
C) MRI screening 3 years postoperatively, then every 2 years thereafter
D) Ultrasonography screening 2 years postoperatively, then every 3 years thereafter
E) Ultrasonography screening 3 years postoperatively, then every 2 years thereafter
The correct response is Option C.
Diagnosis of rupture is difficult by physical examination alone, which is why the majority of ruptures are silent. Subsequent MRI screening for silent rupture is recommended initially 3 years postoperatively, then every 2 years thereafter.
Highly cohesive, anatomically shaped, silicone-filled breast implants combine the “gummy bear” silicone with an anatomical shape, in which inferior pole projection is higher than the superior pole projection. In studies of Allergan’s Natrelle 410 breast implants (the “Pivotal Study,” the 410 Swedish MRI study, and the 410 European MRI study) approximately 3 in 100 women had silent ruptures.
Cohesive gel is still subject to rupture, because rupture occurs when the shell fails. In cohesive implants, however, as opposed to noncohesive implants, the rupture rarely becomes extracapsular.